Potencial gravitacional fuera de los puntos de Lagrange o puntos de Lagrange

El diagrama de ¿Por qué los puntos lagrangianos L4 y L5 son estables? muestra que el potencial gravitatorio disminuye fuera del anillo de puntos de Lagrange; esta imagen lo muestra aún más claramente:Superficie potencial

Si entiendo correctamente, usando la analogía del modelo de lámina de goma, un objeto colocado en el campo se mueve como si una canica rodara sobre la lámina con gravedad hacia abajo. Eso está bien para los objetos dentro del anillo de Lagrange: se mueven hacia cualquier masa.

Pero fuera de ella, implica que se alejan de ambas masas. ¿Es eso realmente lo que sucede? Si es así, ¿por qué? Si no, ¿por qué la superficie se inclina hacia abajo?

¿De dónde sacaste la imagen? Supongo que (como dijo Chay Paterson) la imagen de alguna manera incluye la fuerza de Coriolis así como el potencial gravitacional, pero es difícil dar una respuesta más clara sin un poco más de contexto sobre qué representa exactamente la imagen.
Era de csep10.phys.utk.edu/astr162/lect/binaries/accreting.html aparentemente de una conferencia de la Universidad de Tennessee. El texto que lo acompaña establece específicamente: "La energía potencial gravitatoria para un sistema binario se representa en el diagrama adyacente".
El punto principal es que el potencial centrífugo V C F = 1 2 Ω 2 | z | 2 es un potencial armónico invertido.

Respuestas (2)

Sí, el cuerpo libre se mueve hacia afuera, pero hay dos cosas críticas que debe saber para interpretar esta afirmación correctamente.

Primero, este es el potencial efectivo, teniendo en cuenta la gravedad y la fuerza centrífuga . Tiene esta forma porque entramos en el marco no inercial corrotando con las dos masas. Matemáticamente, el potencial es

Φ mi F F ( r ) = GRAMO ( METRO 1 | r r 1 | potencial de la masa 1 + METRO 2 | r r 2 | potencial de la masa 2 + METRO 1 + METRO 2 2 | r 1 r 2 | 3 | r | 2 componente centrífugo ) ,
y sólo decrece a lo lejos por ese último término.

Físicamente, esto se debe a que colocar un objeto "en reposo" en este marco corresponde a que se mueva con la misma frecuencia angular que METRO 1 y METRO 2 sobre el centro de masa. Si inicializa un objeto 5   A tu en una trayectoria tangencial con la misma velocidad angular que la Tierra, se moverá demasiado rápido para una órbita circular a esa distancia, y por lo tanto se alejará del Sol.

Esto no significa que el objeto desaparecerá para siempre, y eso nos lleva al segundo punto, explicado en la respuesta de Chay: No se han tenido en cuenta todas las fuerzas efectivas; en particular, la fuerza de Coriolis no surge de Φ mi F F . La fuerza de Coriolis depende de la velocidad, por lo que no tiene un potencial escalar que dependa únicamente de la posición, por lo que no se incluye en el análisis hasta el momento. Una vez que su objeto de prueba comience a moverse en su marco giratorio, experimentará una desviación perpendicular que eventualmente lo obligará a girar.

Entonces, ¿qué ilustra el gráfico que se muestra en el OP? ¿Qué pasa con el artículo de Wikipedia? Muestra claramente un campo potencial. Su respuesta parece establecer que esos diagramas con un mínimo / máximo local L2 definitivamente no se muestran Φ mi F F ( r ) , entonces, ¿qué están mostrando? Creo que tus matemáticas muestran este potencial: en.wikipedia.org/wiki/…
Todos estos diagramas son iguales y trazan lo mismo. Φ mi F F Escribi. L4 y L5 son máximos locales, las masas mismas son "mínimos" singulares y L1-3 son todos puntos de silla. Estos diagramas solo establecen el potencial de un objeto corrotante en un radio fijo . Cualquier movimiento con respecto al marco corrotatorio invalida el uso de este potencial (o cualquier escalar) para mostrar movimiento.
Pero en las matemáticas que diste, L4 y L5 no son máximos locales, solo L3 es un máximo. Esta es mi trama de lo que escribiste, es diferente de Wikipedia que tiene L4 y L5 como máximos locales: wolframalpha.com/input/?i=plot+%28-1%2Fsqrt%28x ^2%2By^2%29+- +1%2Fsqrt%28%28x-1%29^2%2Por^2%29+-+%28x^2%2Por^2%29%29+para+x+de+-2+a+2%2C+ y+de+-2+a+2
@AlanSE Ese enlace no funciona, pero creo que veo el problema. Esta fórmula solo es válida si el origen es el centro de masas, por lo que si igualas las dos masas, tienen que estar ubicadas a la misma distancia del centro de masas. y -eje. Prueba una relación de masa de 9:1 con este código:plot (-9/sqrt((x+1/10)^2+y^2) - 1/sqrt((x-9/10)^2+y^2) -5(x^2+y^2)) for x from -2 to 2, y from -2 to 2
Entiendo. Esas matemáticas aclaran el problema matemático.
Muchas gracias, AlanSE. Eso lo explica muy bien. ¡+1 por explicar la ecuación también!

La estabilidad de L4 y L5 no es del todo obvia al observar el potencial gravitacional escalar, porque la estabilidad de las órbitas cerca de esos puntos es muy dinámica: en un marco giratorio como el de la imagen, tienes una velocidad adicional. potencial de Coriolis dependiente, así como el escalar gravitatorio y centrípeto dependiente de la posición ilustrado. Así que aquí hay un segundo potencial, un campo vectorial que va como Ω z ^ (hasta un cartel).

Cuando un objeto es perturbado radialmente desde L4, acelera hacia afuera, pero esto no es lo mismo que moverse hacia afuera monótonamente. Debido a que estamos en un marco giratorio, el movimiento radial da como resultado una aceleración de Coriolis que actúa normalmente a su velocidad, lo que finalmente lo lleva de regreso a donde comenzó.

No conozco ninguna prueba de que esto definitivamente estabilice el problema en el caso de grandes masas de prueba, pero para pequeñas masas de prueba se puede linealizar y demostrar que es dinámicamente estable.

Gracias por la explicación. No me preocupa mucho la estabilidad en L4 o L5, sino las implicaciones en el movimiento lejano. La superficie se inclina hacia abajo (de hecho, a un ritmo creciente) lejos del anillo. ¿Significa eso que un cuerpo libre allí se alejará de las masas?
Tengo problemas con este concepto de "potencial de Coriolis". Para obtener el concepto de potencial en primer lugar, necesita un campo vectorial conservativo. Parece que los campos gravitatorios en este caso claramente no calificarían. Pero la pregunta más importante es ¿cómo se obtiene un potencial de un campo que depende de la velocidad?
@AlanSE: resulta que al igual que el vector potencial en electrodinámica. Vea mi respuesta a la pregunta a la que se hace referencia en esta pregunta.